Difference between revisions of "2005 AMC 12A Problems/Problem 24"

(Problem)
(solution by towersfreak2006)
Line 3: Line 3:
  
 
== Solution ==
 
== Solution ==
{{solution}}
+
Since <math>R(x)</math> has degree three, then <math>P(x)\cdot R(x)</math> has degree six. Thus, <math>P(Q(x))</math> has degree six, so <math>Q(x)</math> must have degree two, since <math>P(x)</math> has degree three.
 +
<div style="text-align:center;">
 +
<math>
 +
P(Q(1))=(Q(1)-1)(Q(1)-2)(Q(1)-3)=P(1)\cdot R(1)=0,
 +
</math><br /><math>
 +
P(Q(2))=(Q(2)-1)(Q(2)-2)(Q(2)-3)=P(2)\cdot R(2)=0,
 +
</math><br /><math>
 +
P(Q(3))=(Q(3)-1)(Q(3)-2)(Q(3)-3)=P(3)\cdot R(3)=0.
 +
</math>
 +
</div>
 +
Hence, we conclude <math>Q(1)</math>, <math>Q(2)</math>, and <math>Q(3)</math> must each be <math>1</math>, <math>2</math>, or <math>3</math>. Since a [[quadratic equation|quadratic]] is uniquely determined by three points, there can be <math>3*3*3 = 27</math> different quadratics <math>Q(x)</math> after each of the values of <math>Q(1)</math>, <math>Q(2)</math>, and <math>Q(3)</math> are chosen.
 +
 
 +
 
 +
However, we have included <math>Q(x)</math> which are not quadratics. Namely,
 +
<div style="text-align:center;">
 +
<math>
 +
Q(1)=Q(2)=Q(3)=1 \Rightarrow Q(x)=1,
 +
</math><br /><math>
 +
Q(1)=Q(2)=Q(3)=2 \Rightarrow Q(x)=2,
 +
</math><br /><math>
 +
Q(1)=Q(2)=Q(3)=3 \Rightarrow Q(x)=3,
 +
</math><br /><math>
 +
Q(1)=1, Q(2)=2, Q(3)=3 \Rightarrow Q(x)=x,
 +
</math><br /><math>
 +
Q(1)=3, Q(2)=2, Q(3)=1 \Rightarrow Q(x)=4-x.
 +
</math>
 +
</div>
 +
Clearly, we could not have included any other constant functions. For any linear function, we have <math>2\cdot Q(2) = Q(1) + Q(3)</math>. Again, it is pretty obvious that we have not included any other linear functions. Therefore, the desired answer is <math>27 - 5 = 22</math>.
 +
 
 
== See also ==
 
== See also ==
* [[2005 AMC 12A Problems/Problem 23 | Previous problem]]
+
{{AMC12 box|year=2005|ab=A|num-b=23|num-a=25}}
* [[2005 AMC 12A Problems/Problem 25 | Next problem]]
+
 
* [[2005 AMC 12A Problems]]
+
[[Category:Intermediate Algebra Problems]]

Revision as of 22:16, 22 September 2007

Problem

Let $P(x)=(x-1)(x-2)(x-3)$. For how many polynomials $Q(x)$ does there exist a polynomial $R(x)$ of degree 3 such that $P(Q(x))=P(x)* R(x)$?

Solution

Since $R(x)$ has degree three, then $P(x)\cdot R(x)$ has degree six. Thus, $P(Q(x))$ has degree six, so $Q(x)$ must have degree two, since $P(x)$ has degree three.

$P(Q(1))=(Q(1)-1)(Q(1)-2)(Q(1)-3)=P(1)\cdot R(1)=0,$
$P(Q(2))=(Q(2)-1)(Q(2)-2)(Q(2)-3)=P(2)\cdot R(2)=0,$
$P(Q(3))=(Q(3)-1)(Q(3)-2)(Q(3)-3)=P(3)\cdot R(3)=0.$

Hence, we conclude $Q(1)$, $Q(2)$, and $Q(3)$ must each be $1$, $2$, or $3$. Since a quadratic is uniquely determined by three points, there can be $3*3*3 = 27$ different quadratics $Q(x)$ after each of the values of $Q(1)$, $Q(2)$, and $Q(3)$ are chosen.


However, we have included $Q(x)$ which are not quadratics. Namely,

$Q(1)=Q(2)=Q(3)=1 \Rightarrow Q(x)=1,$
$Q(1)=Q(2)=Q(3)=2 \Rightarrow Q(x)=2,$
$Q(1)=Q(2)=Q(3)=3 \Rightarrow Q(x)=3,$
$Q(1)=1, Q(2)=2, Q(3)=3 \Rightarrow Q(x)=x,$
$Q(1)=3, Q(2)=2, Q(3)=1 \Rightarrow Q(x)=4-x.$

Clearly, we could not have included any other constant functions. For any linear function, we have $2\cdot Q(2) = Q(1) + Q(3)$. Again, it is pretty obvious that we have not included any other linear functions. Therefore, the desired answer is $27 - 5 = 22$.

See also

2005 AMC 12A (ProblemsAnswer KeyResources)
Preceded by
Problem 23
Followed by
Problem 25
1 2 3 4 5 6 7 8 9 10 11 12 13 14 15 16 17 18 19 20 21 22 23 24 25
All AMC 12 Problems and Solutions